6 13 51
4 7 29
19 11 ?
What the missing number?

Answers

Answer 1

Answer:

I think the answer is 3 ( Maybe)


Related Questions

Limes are on sale. That sale price is 8 limes for $2.00. Why could the unit rate be 4 or 0.25?

Answers

Answer:

 C.No, because each lime will cost a bit more than 30¢, so 4 limes will cost a bit more than $1.20.Step-by-step explanation:

The unit price of the 8 limes is $0.25 per lime.

The given expression:

The selling price for 8 limes = $2.00

To find:

if the unit price is $4 or $0.25

The unit price of the lime is calculated by dividing the total selling price by the total number of limes purchased.

[tex]unit \ price = \frac{total \ amount \ paid \ for \ 8 \ limes }{8 \ limes } \\\\unit \ price =\frac{\$ \ 2}{8 \ limes } \\\\unit \ price = (\frac{1}{4} ) \frac{\$}{lime} \\\\unit \ price = \$0.25 \ per \ lime[/tex]

Thus, the unit price of the 8 limes is $0.25 per lime.

Learn more here: https://brainly.com/question/12418981

Minstrel Manufacturing uses a job order costing system. During one month, Minstrel purchased $198,000 of raw materials on credit; issued materials to production of $195,000 of which $30,000 were indirect. Minstrel incurred a factory payroll of $150,000, of which $40,000 was indirect labor. Minstrel uses a predetermined overhead rate of 150% of direct labor cost. The total manufacturing costs added during the period is:

Answers

Answer:

$440,000

Step-by-step explanation:

Direct material:

= $195,000 - $30,000

= $165,000

Direct labor:

= $150,000 - $40,000

= $110,000

Manufacturing overhead:

150% of direct labor cost.

= $110,000 x 150 ÷ 100

= $16,500,000 ÷ 100

= $165,000

Total manufacturing costs:

= $165,000 + $110,000 + $165,000

= $275,000 + $165,000

= $440,000

The total manufacturing costs added during the period is: $440,000

15. Bradley has a goal to work 28 hours each week at the pizza shop. So far he has
worked 12 hours. How many more hours does he need to work to meet his goal?
70

Answers

16

28-12=16

He needs 16 more hours of work to complete

a 3 yard roll of waxed paper costs $3.33. what is the price per foot?

Answers

Answer:

$.37 per foot

Step-by-step explanation:

1 yd = 3ft

3 yd = 3*3 = 9ft

Take the dollar amount and divide by the number of ft

3.33 / 9

$.37 per foot

Answer:

$0.37 per foot

Step-by-step explanation:

3 feet=1 yard 3 yards= 9 feet

First Way:

3.33 divided by 9 = 0.37

Second Way:

9x= 3.33

9x÷9. 3.33÷ 9

x= 0.37 per foot

A parallelogram has an area of 20.4 square units. If the height that corresponds to a base is 4 units,what is the base

Answers

Answer:

5.1 units

Step-by-step explanation:

A=b x h divide by 2

20.4=4b

b=5.1 units

As a surveyor, you measure an angle of elevation as 35.25 degrees. You are required to record all measurements in fraction form. What is the degree measure of the angle written as a fraction?

Answers

Answer:

35 1/4

Step-by-step explanation:

Answer:

B for edge

Step-by-step explanation:

If x=4 what is the value of 2x+18

Answers

Answer:

26

Step-by-step explanation:

2x + 18

2(4) + 18

8 + 18

26

Answer:

26

Step-by-step explanation:

We just substitute the value of X and find the answer

4(x - 3) = 4(2x + 1)

Answers

Hi there! :)

Answer:

[tex]\huge\boxed{x = -4}[/tex]

4(x - 3) = 4(2x + 1)

Distribute the coefficient outside of the parenthesis:

4(x)+ 4(-3) = 4(2x) + 4(1)

Simplify:

4x - 12 = 8x + 4

Subtract 4x from both sides:

4x - 4x - 12 = 8x - 4x + 4

-12 = 4x + 4

Subtract 4 from both sides:

-12 -4 = 4x + 4 - 4

-16 = 4x

Divide both sides by 4:

-16/4 = 4x/4

x = -4.

Answer:

x=-4

Step-by-step explanation:

or 4*×-4*3=4*2x+4*1

or 4x-12=8x+4

or 4x-8x=4+12

or -4x=16

or x=16/-4

or =x=-4

Suppose a deep sea diver dives from the surface to 202 feet below the surface. He then dives down 12 more feet. Use integers to represent this situation. Then find
the diver's present depth.
Which expression best represents the diver's situation.
O A. O + (-202) + 12
OB. 0 + (-202)+(-12)
OC. 0+202 +(-12)
OD. 0+202 + 12
The diver is presently feet below the surface.
(Simplify your answer.)

Answers

Answer:

B.) (-202)+(-12)

Step-by-step explanation:

The undersea level is negative then going under again will produce a completely negative answer.

Please help me solve this 166=-w+66

Answers

Answer:

w = -100

Step-by-step explanation:

Step 1: Write out equation

166 = -w + 66

Step 2: Subtract 66 on both sides

100 = -w

Step 3: Divide both sides by -1

w = -100

Answer:

w=-100

Step-by-step explanation:

We are given the equation:

166= -w +66

To solve for x, we must get x by itself on one side of the equation.

66 is being added to -w. The inverse of addition is subtraction. Subtract 66 from both sides of the equation.

166-66= -w+66-66

166-66= -w

100= -w

-1 and w are being multiplied. The invers of multiplication is division. Divide both sides of the equation by -1.

100/-1= -w/-1

100/-1=w

-100=w

Let's check our solution. Plug -100 in for w.

166= -w+66

166= -(-100)+66

166=100+66

166=166

The statement above is true, so we know our solution is correct.

The solution to 166= -w+66 is w= -100

Roll three fair 6-sided dice. What is the chance that they will all land on ace? What is the chance that they will not all land on ace? What is the chance that none of them will land on ace ?

Answers

Answer:

5% chance of ace and 15% of not landing ace

Step-by-step explanation:

lol trust me i play dice in school

How do you solve this equation by using the quadratic formula 8x^2+3x-45=0

Answers

The 8 is a, the 3 is b and -45 is c

Plug them into the quadratic formula

How many whole tens are in 3,200

Answers

Answer:

320

Step-by-step explanation:

There are 320 tens in 3,200.

320 x 10 = 3200

Answer:

320

Step-by-step explanation:

3200/10=320

Answer 9w-4=14 show solving steps pls

Answers

Answer:

[tex]w=2[/tex]

Step-by-step explanation:

So we have the equation:

[tex]9w-4=14[/tex]

Add 4 to both sides. The left cancels:

[tex](9w-4)+4=(14)+4\\9w=18[/tex]

Divide both sides by 9.The left cancels:

[tex](9w)/9=(18)/9\\w=2[/tex]

So, the value of w is 2.

And we're done :)

In a recent survey, three out of every five students said they would prefer going to a water park for the class trip.
If 105 students were surveyed, how many can be expected to prefer the water park?
O A 21
OB. 35
O c. 63
O D. 70
E. 103

Answers

Answer:

the answer is d

Step-by-step explanation:

Just multiply and divide

What is probability?

the quality or state of being probable; the extent to which something is likely to happen or be the case

We have,

Surveyed student = 105

Students would prefer to going to a water park= 3/5

According to question

students who expected to prefer the water park

                 = 105 × 3/5

                 = 21×5×3/5

                 = 21×3

                 = 63

Hence, students who expected to prefer the water park is 63

To learn more about probability from here

https://brainly.in/question/20635873

#SPJ2

the height of the house is 26 feet what is the height x of each story?

Answers

Answer:

1 story=26, 2 stories=13 3 stories= 8.666666 4 stories= 6.5 5 stories=5.2

Step-by-step explanation:

A stock price S is governed where z is a standardized Wiener process. Find the process that governs G(t)

Answers

This question is incomplete, the complete question is;

A stock price S is governed by dS = aSdt + bSdz

where z is a standardized Wiener process. Find the process that governs G(t) = S^1/2(t)

Answer:

G = S^1/2

Step-by-step explanation:

Solving the Equation

dS = aSdt + bSdz

First we Take S common from Right hand Side

dS = S(a dt + b dz)

Then we also take S Left Hand Side(LHS) from RHS

dS/S = a dt + b dz

So d = a dt + b dz

now we Take d Common from RHS

d = d(a t + b z)

So

d/d = a t + b z

1 = a t + b z

So, t = (1-b z) / a

Now  substitute value of t in equation G(t) = S^1/2(t)

we have

G{(1- b z)/a} = S^1/2 {(1- b z)/a}

(1- b z)/a) from both sides cancels out each other

So we have  G = S^1/2

Amber owns a plum orchard, and needs to harvest at least 2293 plums to cover the costs of running the orchard, but also wants to donate 280 plums to charity. If each tree bears an average of 83 plums, how many plum trees need to be in the orchard? Express your answer in interval notation.

Answers

Answer:

27 trees needs to be in the orchard.

Step-by-step explanation:

amber needs to harvest at least 2293 of plums to cover the cost of running the orchard.

but also wants to donate 280 plums.

each tree produces an average of 83 plums

find: how many plums of trees needed to be in the orchard?

number of trees required = total harvest / number of plums produced per tree

therefore,

number of trees required = 2293 / 83 ≅ 27 trees needs to be in the orchard.

so Amber can donate 280 plums to charity.

Given the figure below, find the values of x and z

Answers

20x-100=180
x=14
12(14)-60
108
180-108
z=72

Answer:

x=14 z=72

Step-by-step explanation:

(8x-40)+(12x-60)=180

20x-100=180

20x=280

x=14

(12x-60)+z=180

(168-60)+z=180

108+z=180

z=72

Simplify: 3(2x-y)-(5x+4y-2)
Please show steps

Answers

Answer:

6 x ^2 − 5 x y − 4 y ^2

Step-by-step explanation:

Expand  

( 2 x + y ) (3 x − 4 y )

using the FOIL Method.

Apply the distributive property.

2 x ( 3 x − 4 y ) + y ( 3 x − 4 y )

Apply the distributive property.

2 x ( 3 x )+ 2 x (− 4 y ) + y ( 3x − 4 y )

Apply the distributive property.

2 x ( 3 x ) + 2 x( − 4 y ) + y ( 3 x) + y ( −4 y )

Simplify and combine like terms.

6 x ^2 − 5 x y − 4^ 2

Step-by-step explanation:

so it is 3×2=6 3×y=3y so here it 6-3 =3

5+4=9-2=7

so 7-3=4

What is an equation for "five more than the
product of 7 and a number t is 10?"

Answers

Answer:

Option (B) will be the correct option.

Step-by-step explanation:

Statement states "five more than the product of seven and a number t is 10."

Split this statement into parts.

1). Product of 7 and a number 't' = 7 × t

2). 5 more than the product of 7 and a number 't' = 7t + 5

3). Five more than the product of 7 and a number t is equal to 10 ⇒ 7t + 5 = 10

Therefore, Option (B) will be the correct option.

I need help with the following question, whichever one is right will get brainliest!

Answers

Given: m∠A + m∠B = 90° (Definition of Complementary Angles).

Given: m∠B = (5x + 8)°

A) m∠A = 180° + (5x + 8) is your answer choice:

Why it isn't the others:

B) m∠A = 90° - (5x + 8)

You are solving for complementary angles (90° in total when combined), so you subtract the measurement for B (5x + 8) to get A.

C) m∠A = 180° - 2(5x + 8)°

You double the complementary angle to solve for the supplementary. Essentially, just divide this answer by two to get the complementary angles.

D) m∠A = 82 - 5x

I was confused by this one, but essentially they just did one step of the isolating the variable, which was subtracting 8 from both sides. They did not finish isolating the variable however.

Find the value of x in the triangle shown below.
12

Answers

Answer:

The answer is option C

Step-by-step explanation:

Since the triangle is a right angled triangle we can use Pythagoras theorem to find the missing side x

Using Pythagoras theorem we have

c² = a² + b²

where

c is the hypotenuse

From the question x is the hypotenuse

So we have

[tex] {x}^{2} = {5}^{2} + {12}^{2} \\ x = \sqrt{25 + 144} \\ x = \sqrt{169} [/tex]

We have the final answer as

x = 13

Hope this helps you

the red line in the figure is an altitude of triangle ABC. Using right angle trigonometry, write an equation involving sin C​

Answers

Answer:

[tex]\Large \boxed{\mathrm{\bold{C}}}[/tex]

Step-by-step explanation:

[tex]\sf \displaystyle sin(\theta)=\frac{opposite}{hypotenuse}[/tex]

The side opposite to [tex]\angle C[/tex] is h.

The hypotenuse of the smaller right triangle is a.

[tex]\sf \displaystyle sin(C)=\frac{h}{a}[/tex]

Answer:

sinC = [tex]\frac{h}{a}[/tex]

Step-by-step explanation:

Since the red line is an altitude then the triangles are right, thus

sinC = [tex]\frac{opposite}{hypotenuse}[/tex] = [tex]\frac{h}{a}[/tex]

1. Graph the line y=2x+5 on the graph. ​

Answers

Answer:

See below

Step-by-step explanation:

We can simply use a graphing calculator to carry it out.

See the attached file for more explanation!

Another way is that we can take the values of x as 1,2 and 3 and so on and put it in the equation to get the value of y. We have some coordinates now so we'll plot them in the graph to get the line of y = 2x+5.

Step-by-step explanation:

Hope this helps........

what is the probability that a randomly selected driver fatality who was female was 55 to 69 years old

Answers

Answer:

0.1354

Step-by-step explanation:

Relevant data provided as per the requirement is shown below:-

Female probability of age between 55 to 69 = 2058

Male probability of age between 55 to 69 = 4571

According to the given situation, the calculation of probability is shown below:-

[tex]= \frac{Female\ from\ 55\ to\ 69 }{Total\ female}[/tex]

where,

Total female is

= 143 + 2333 + 4027 + 5178 + 2058 + 1459

= 15,198

And, the female is 2058

So, the probability is

[tex]= \frac{2058}{15,198}[/tex]

= 0.1354

Therefore for computing the probability of female that lies between 55 to 69 we simply applied the above formula.

New heat lamps are reported to have the mean lifespan of 100 hours with a standard deviation of 15 hours. Before replacing their current lamp to the new heat lamps for the university, OSU decided to test whether the mean lifetime is equal to 100 or not by sampling 36 heat lamps. They turned them on and recorded the time, in hours, until each lamp failed. The sample provided a mean lifespan is 105.1 hours.
1) What set of hypotheses are correct for this problem?
SET 1 - H0: µ = 100 hours , Ha: µ < 100 hours
SET 2 - H0: µ = 100 hours , Ha: µ > 100 hours
SET 3 - H0: µ = 100 hours , Ha: µ ≠ 100 hours
A) SET 1.
B) SET 2.
C) SET 3.
2) If we assume the null hypothesis to be true, the average of the distribution of sample means, μ x ¯, from a sample size of 36 is:______.
a) 15.
b) 115.
c) 100.
d) 105.1. .
3) According to the Central Limit Theorem, the standard deviation of the distribution of the sample means is:______.
a) 115.
b) 15.
c) 6.
d) 2.5. .
4) What is the approximate probability of observing a sample mean of 105.1 or more from the distribution of sample means, again assuming that the null hypothesis is true?
a) 0.68.
b) 0.025.
c) 0.975.
d) 0.16.

Answers

Answer:

1

  The  correct option is  C

2

 The  correct option is C

3

The  correct option is A

4

The  correct option is B

Step-by-step explanation:

From the question we are told that

    The  population mean is  [tex]\mu = 100[/tex]

     The  standard deviation is  [tex]\sigma = 15[/tex]

     The  sample size is  [tex]n = 36[/tex]

     The  sample mean is  [tex]\= x = 105.1[/tex]

Generally

    The  null hypothesis is  [tex]H_o: \mu = 100 \ hours[/tex]

     The  alternative hypothesis is  [tex]H_a : \mu \ne 100\ hours[/tex]

Given that the null hypothesis is  true then the  distribution of sample means  [tex]\mu_{\= x }[/tex], from a sample size of 36 is  mathematically represented as

        [tex]\mu_{\= x } = \mu[/tex]

=>     [tex]\mu_{\= x } = 100[/tex]

According to the Central Limit Theorem the test stated in the question is  approximately normally distributed if the sample size is sufficiently large[tex](n > 30 )[/tex] so given that the sample size is  large  n =  36

Then the test is normally distributed and hence the standard deviation is 15

Generally the standard error of mean is mathematically represented as

     [tex]\sigma_{\= x } = \frac{ \sigma }{\sqrt{n} }[/tex]

=>  [tex]\sigma_{\= x } = \frac{15}{\sqrt{36} }[/tex]

=>  [tex]\sigma_{\= x } = 2.5[/tex]

Generally  the approximate probability of observing a sample mean of 105.1 or more is mathematically represented as

        [tex]P( \= X \ge 105.1 ) =1 - P(\= X < 105.1) = 1- P(\frac{\= X - \mu }{\sigma_{\= x }} <\frac{105.1 - 100}{2.5} )[/tex]

=>    [tex]P( \= X \ge 105.1 ) =1 - P(\= X < 105.1) = 1- P(Z<2.04 )[/tex]

From the z-table (reference calculator dot net )

             [tex]P(Z<2.04 ) = 0.97932[/tex]

So

  [tex]P( \= X \ge 105.1 )= 1 - P(\= X < 105.1) = 1- 0.97932[/tex]

 [tex]P( \= X \ge 105.1 ) =0.02[/tex]    

what is the annual simple interest rate?
I=$17, P=$500, t=2 years​

Answers

Answer:

[tex] \boxed{ \bold{ \huge{ \boxed{ \sf{1.7 \: \%}}}}}[/tex]

Step-by-step explanation:

Given,

Interest ( I ) = $ 17

Principal ( P ) = $ 500

Time ( T ) = 2 years

Rate ( R ) = ?

Finding the simple Interest rate :

[tex] \boxed{ \bold{ \sf{rate = \frac{interest \times 100}{principal \times time}}}} [/tex]

[tex] \dashrightarrow{ \sf{rate = \frac{17 \times 100}{500 \times 2} }}[/tex]

[tex] \dashrightarrow{ \sf{rate = \frac{1700}{1000} }}[/tex]

[tex] \dashrightarrow{ \sf{ rate = 1.7 \: \%}}[/tex]

Hope I helped!

Best regards! :D

Given the sets
A
and
B
expressed in interval notation, find
A

B
.

A
=
(


,

42
)

(

25
,
+

)


B
=
(

54
,
70
)

Answers

Answer:

Step-by-step explanation:

A∩B=[-54,-42]∪[-25,70]

A fruit bowl has 5 apples 7 oranges and 4 bananas. What is the ratio of apples to bananas?

Answers

Answer:

5 apples to 4 bananas

Step-by-step explanation:

Answer:

5:4

Step-by-step explanation:

A ratio divides the number with a colon. And since there are 5 apples to 4 bananas, the ratio of apples to bananas would be 5:4 (or another way to write it is 5 to 4)

Other Questions
plz reanswer it If (x+4): (3x+1) is the duplicate ratio of 3:4 find the value of x. Solve 3(x + 1) + 6 = 33. At 5:00 a.M., a parking lot had 16 cars already parked in it. Throughout the course of the day, 48 more cars entered and parked in the parking lot. Which situation would result in a difference of 0 for the cars exiting the lot for the day? If I had 39inches and 28inches how many inches and all All chemicals in the lab (including foodstuffs and store-bought chemicals) should be treated as if they could be hazardous. True or False? The following image represents the correct way to name ____* which qualities should a good neighbour have?? Megan expects to save an average of 5 dollars perweek from her babysitting earnings. The relationshipbetween time and the amount of money she saves isshown on the graph. Which other ordered pairs wouldfall on this line? Select all that apply.A. (0.5, 2.5)B. (3, 18)C. (0,0)D. (4.5, 30) When comparing the productivity of an employee with the hours that the employee works, the conclusion is that "there is _____" between these variables. Which of Barbara Frietchies actions leads most directly to a conflict? Select the correct answer.You travel in a circle, whose circumference is 8 kilometers, at an average speed of 8 kilometers/hour. If you stop at the same point you startedfrom, what is your average velocity?OA. O kilometers/hour. 2 kilometers/hourOC. 4 kilometers/hourOD. 8 kilometers/hour . 16 kilometers/hour Why is it important to have good communication skills? Write a piecewise function. PLEAE HELP i really need this any help would be very appreciated!! Either Of the boys are coming today. Find The Error A 11,000-kg train car moving due east at 21.0 m/s collides with and couples to a 23,000-kg train car that is initially at rest. What is the common velocty of the two-car train after the collisions? Find the area.7 cm17 cm25 cm Cancer and diabetes are two common hereditary diseases. Radocarbon dating was devolped in 1949 by Blast From The Past! Why did the New England & Middle Regions not depend on slavery? Fill in the blank: The ability of a group of organisms to change, or ______time helps populations survive in an ever-changing environment. *A. HomeostasisB. MetabolismC. StimulusD. Evolve